Sequences and Roots!

Calculus Level 5

P ( x ) = m = 0 n F m x m \displaystyle P(x) = \sum_{m=0}^n F_mx^m

Let P ( x ) P(x) be an n t h n^{th} degree polynomial defined as above for a certain odd natural number n n . And denote by { r 1 , r 2 , r 3 , , r n } \{r_1,r_2,r_3,\dots, r_n \} as the set of roots of P P .

Let a k {a_k} be a sequence defined by:

1 a k r k a k 1 = r k r k + 1 r n \displaystyle \frac{1}{a_k-r_ka_{k-1}} = r_{k}r_{k+1}\dots r_n

k n , k N \forall k\leq n , k\in N^* , with a 0 = 1 a_0=1

Find:

lim n ( a n + 1 ) F n + 1 \displaystyle \lim_{n\to \infty} (a_n+1)F_{n+1}

D e t a i l s a n d A s s u m p t i o n s : \mathbf{\; Details \; and \; Assumptions \; :}

{ F m } \bullet \{F_m\} denotes the Fibonacci Sequence ( F 0 = F 1 = 1 F_0=F_1=1 ).

\bullet The set of roots of P P also takes into consideration the multiplicity (for example, double roots exist twice).

Try my other calculus challenges here


The answer is -1.618.

This section requires Javascript.
You are seeing this because something didn't load right. We suggest you, (a) try refreshing the page, (b) enabling javascript if it is disabled on your browser and, finally, (c) loading the non-javascript version of this page . We're sorry about the hassle.

1 solution

Hasan Kassim
Mar 7, 2015

Rearranging the given recursive formula:

a k = r k a k 1 + 1 j = k n r j \displaystyle a_k=r_ka_{k-1} + \frac{1}{\prod_{j=k}^nr_j}

In general:

a k = B k a k 1 + M k \displaystyle a_k=B_ka_{k-1} + M_k

= B k B k 1 a k 2 + M k 1 B k + M k \displaystyle = B_kB_{k-1}a_{k-2} + M_{k-1}B_k +M_k

= B k B k 1 B k 2 a k 3 + M k 2 B k B k 1 + M k 1 B k + M k \displaystyle = B_kB_{k-1}B_{k-2}a_{k-3} + M_{k-2}B_kB_{k-1} + M_{k-1}B_k +M_k

= = \dots

Recognizing the pattern, we will Propose then show by induction (at the end of the solution) that:

a k = ( p = 1 k B p ) ( a 0 + i = 1 k M i t = 1 i B t ) \displaystyle a_k = \Bigg(\prod_{p=1}^k B_p \Bigg) \Bigg( a_0 +\sum_{i=1}^k \frac{M_i}{\prod_{t=1}^i B_t} \Bigg)

Substitute: a 0 = 1 , B k = r k a_0=1 , B_k = r_k and M k = 1 j = k n r j M_k = \frac{1}{\prod_{j=k}^nr_j} :

a k = ( p = 1 k ( r p ) ) ( 1 + i = 1 k 1 j = i n r j t = 1 i ( r t ) ) \displaystyle a_k = \Bigg(\prod_{p=1}^k (r_p) \Bigg) \Bigg( 1 +\sum_{i=1}^k \frac{\frac{1}{\prod_{j=i}^nr_j}}{\prod_{t=1}^i (r_t)} \Bigg)

= ( p = 1 k r p ) ( 1 + i = 1 k 1 r i j = 1 n r j ) \displaystyle = \Bigg(\prod_{p=1}^k r_p \Bigg) \Bigg( 1 +\sum_{i=1}^k \frac{1}{r_i\prod_{j=1}^n r_j} \Bigg)

Let k = n k=n :

a n = ( p = 1 n r p ) ( 1 + i = 1 n 1 r i j = 1 n r j ) \displaystyle a_n= \Bigg(\prod_{p=1}^n r_p \Bigg) \Bigg( 1 +\sum_{i=1}^n \frac{1}{r_i\prod_{j=1}^n r_j} \Bigg)

Note that { r 1 , r 2 , , r n } \{r_1,r_2,\dots,r_n \} are the roots of P ( x ) = m = 0 n F m x m P(x) = \sum_{m=0}^n F_mx^m . Hence by Vieta's formula :

j = 1 n r j = ( 1 ) n F 0 F n = 1 F n ( n is odd ) \displaystyle \prod_{j=1}^n r_j = (-1)^n\frac{F_0}{F_n} = - \frac{1}{F_n} \;\;\;\; ( n \textrm{ is odd})

= > a n = 1 F n i = 1 n 1 r i F n \displaystyle => a_n = -\frac{1-F_n \sum_{i=1}^n \frac{1}{r_i} }{F_n}

Now working on the sum :

i = 1 n 1 r i \displaystyle \sum_{i=1}^n \frac{1}{r_i}

= i = 1 n j = 1 n r j r i j = 1 n r j \displaystyle = \frac{\sum_{i=1}^n \frac{\prod_{j=1}^n r_j}{r_i}}{\prod_{j=1}^n r_j}

= c y c r 1 r 2 r n 1 j = 1 n r j \displaystyle = \frac{\sum_{cyc} r_1r_2\dots r_{n-1} }{\prod_{j=1}^n r_j}

Using Vieta again :

c y c r 1 r 2 r n 1 = ( 1 ) n 1 F 1 F n = 1 F n \displaystyle \sum_{cyc} r_1r_2\dots r_{n-1} = (-1)^{n-1}\frac{F_1}{F_n} = \frac{1}{F_n}

= > i = 1 n 1 r i = 1 F n 1 F n = 1 \displaystyle => \sum_{i=1}^n \frac{1}{r_i} = \frac{\frac{1}{F_n} }{-\frac{1}{F_n} } = -1

Substitute back in a n a_n :

a n = 1 + F n F n \displaystyle \boxed{ a_n = -\frac{1+F_n}{F_n} }

Now perform the recommended limit:

lim n ( a n + 1 ) F n + 1 \displaystyle \lim_{n\to \infty} (a_n+1)F_{n+1}

= lim n F n + 1 F n \displaystyle = - \lim_{n\to \infty} \frac{F_{n+1}}{F_n}

= ϕ \displaystyle =\boxed{ -\phi}

Where ϕ \phi is the Golden Ratio .(The limit is proved below)

. . . . . . . . . . . . . . . . . . . . . . . . . . . . . . . . . . . . ....................................

T h e I n d u c t i o n p r o o f : \bullet \mathbf{ \;The \; Induction \; proof \; :}

For a k = B k a k 1 + M k a_k=B_ka_{k-1} + M_k , we have:

a k = ( p = 1 k B p ) ( a 0 + i = 1 k M i t = 1 i B t ) \displaystyle a_k= \Bigg(\prod_{p=1}^k B_p \Bigg) \Bigg( a_0 +\sum_{i=1}^k \frac{M_i}{\prod_{t=1}^i B_t} \Bigg)

\ast Test for k = 1 k=1 :

a 1 = ( p = 1 1 B p ) ( a 0 + i = 1 1 M i t = 1 i B t ) \displaystyle a_1= \Bigg(\prod_{p=1}^1 B_p \Bigg) \Bigg( a_0 +\sum_{i=1}^1 \frac{M_i}{\prod_{t=1}^i B_t} \Bigg)

a 1 = ( B 1 ) ( a 0 + M 1 B 1 ) = B 1 a 0 + M 1 \displaystyle a_1 =( B_1 )( a_0 +\frac{M_1}{B_1} )= B_1a_0 + M_1

Which is true by setting k = 1 k=1 in the recursive formula.

\ast Suppose that it is true for k k , Test for k + 1 k+1 :

a k + 1 = ( p = 1 k + 1 B p ) ( a 0 + i = 1 k + 1 M i t = 1 i B t ) \displaystyle a_{k+1} = \Bigg(\prod_{p=1}^{k+1} B_p \Bigg) \Bigg( a_0 +\sum_{i=1}^{k+1} \frac{M_i}{\prod_{t=1}^i B_t} \Bigg)

= B k + 1 ( p = 1 k B p ) ( a 0 + i = 1 k M i t = 1 i B t + M k + 1 t = 1 k + 1 B t ) \displaystyle = B_{k+1} \Bigg(\prod_{p=1}^k B_p \Bigg) \Bigg( a_0 + \sum_{i=1}^k \frac{M_i}{\prod_{t=1}^i B_t} +\frac{M_{k+1}}{\prod_{t=1}^{k+1} B_t} \Bigg)

= B k + 1 ( p = 1 k B p ) ( a 0 + i = 1 k M i t = 1 i B t ) + M k + 1 \displaystyle = B_{k+1} \Bigg(\prod_{p=1}^k B_p \Bigg) \Bigg( a_0 + \sum_{i=1}^k \frac{M_i}{\prod_{t=1}^i B_t} \Bigg) + M_{k+1}

= B k + 1 a k + M k + 1 \displaystyle = B_{k+1}a_k + M_{k+1}

Which is true by the recursive formula, Therefore our preposition is validated.

P r o o f o f T h e L i m i t : \bullet \mathbf{ \; Proof \; of \; The \; Limit \; : }

L = lim n F n + 1 F n \displaystyle L= \lim_{n\to \infty} \frac{F_{n+1}}{F_n}

Use the recursive formula of the Fibonacci sequence:

F n + 1 = F n + F n 1 \displaystyle F_{n+1} = F_{n} +F_{n-1}

= > L = lim n F n + F n 1 F n = 1 + 1 lim n F n F n 1 = 1 + 1 L \displaystyle => L = \lim_{n\to \infty} \frac{F_{n} +F_{n-1}}{F_n} = 1+ \frac{1}{ \lim_{n\to \infty} \frac{F_n}{F_{n-1}} } = 1+\frac{1}{L}

Solving for L L yields the Golden Ratio .

Nice solution

Uchiha Robert - 6 years, 3 months ago

0 pending reports

×

Problem Loading...

Note Loading...

Set Loading...